IMO Shortlist 2004 problem A2


Kvaliteta:
  Avg: 4,5
Težina:
  Avg: 6,0
Dodao/la: arhiva
2. travnja 2012.
LaTeX PDF
Let a_0, a_1, a_2, ... be an infinite sequence of real numbers satisfying the equation a_n=\left|a_{n+1}-a_{n+2}\right| for all n\geq 0, where a_0 and a_1 are two different positive reals.

Can this sequence a_0, a_1, a_2, ... be bounded?

Remark This one is from the IMO Shortlist 2004, but it's already published on the official BWM website und thus I take the freedom to post it here:
Izvor: Međunarodna matematička olimpijada, shortlist 2004